Triple Integral (Calc III-Multivariable Calc)

Click For Summary
The discussion focuses on integrating the function f(x,y,z)=3x+8y over a solid defined by a slice of an ice cream cone in the first octant, bounded by specific planes and contained within a sphere and a cone. Participants clarify the need to use both Cartesian and potentially spherical coordinates to visualize the integration bounds. The solid's integration limits are debated, particularly how to represent the intersection of the cone and sphere. Suggestions include sketching the shapes for better understanding and using computational tools for visualization. The conversation emphasizes the importance of correctly identifying the bounds for the triple integral to solve the problem effectively.
MhailJ
Messages
1
Reaction score
0

Homework Statement



Integrate the function f(x,y,z)=3x+8y over the solid given by the "slice" of an ice-cream cone in the first octant bounded by the planes x=0 and y=(sqrt(17/47))*x and contained in a sphere centered at the origin with radius 10 and a cone opening upwards from the origin with top radius 8.


Homework Equations



I know that we will use both the planes, x=0 and y=sqrt17/47. As well as the sphere (x^2+y^2+z^2=10) and the cone (x^2+y^2=8z^2).

The Attempt at a Solution



I understand that this shape will look something like an ice cream cone, with a plan in the yz-plane and the other given plane as bounds. However, I do not understand what bounds go where. Due to the fact that it is a "slice" of an ice cream cone, does this mean spherical coordinates must be used to solve this equation? I do not quite understand what solid they want to integrate with respect to. Does this mean that the solid is IN the cone? If so, why do they give me the sphere that the cone is in- because the top of the cone ends before the sphere ends, therefore giving the "ice cream cone" type look to this figure? If someone could just give me a start with this problem in terms of what it looks like I think I can work out the rest...I know this is quite difficult to show online due to the fact that I cannot show you guys my sketch that I currently have done.

Thank you very much!
MhailJ
 
Physics news on Phys.org
It looks barely like some quadrant of the cone. Also, if the radius of the sphere is 10, then that would be x^2+y^2+z^2=100 right? But it won't qualitatively change the problem. Here's what I suggest: It's good to try and draw them free-hand . . . for a little while, then learn how to do them in a CAS like Mathematica. The plot below is what I did so I assume it's over that green piece in there. Red is the cone, blue is the paraboloid, green the sphere and dark-green x=0. Alright, assume that's what you want. Now, just for starters, do it in cartesian coordinates even though that will probably be too complex to integrate but that at least will help you visualize how to integrate it. So, consider:

\int\int\int f(x,y,z) dzdydx

so at least conceptually, the dz is easy right? It's just in that green piece starting from the cone, up to the sphere:

\int\int\int_{red}^{light green} f(x,y,z) dzdydx

I'll do the red one for you. If it's equation is x^2+y^2=8z^2[/tex], then my lower bound on z would be:<br /> <br /> \sqrt{1/8(x^2+y^2)}<br /> <br /> You do the other one. Now what about y? Isn&#039;t that going from the border of the blue paraboloid to the border where the cone and sphere intersect? So the lower bound on y would be just the equation of the paraboloid in the x-y plane. You try and figure where the equations x^2+y^2+z^2=10[/tex] and x^2+y^2=8z^2[/tex] interesect for the top limit. So far then we have:&amp;lt;br /&amp;gt; &amp;lt;br /&amp;gt; \int_a^b \int_{\sqrt{17/47 x}}^{myintersection}\int_{1/8(x^2+y^2)}^{mysphere} f(x,y,z)dzdydx&amp;lt;br /&amp;gt; &amp;lt;br /&amp;gt; Now you try and finish it.
 

Attachments

  • mycone.jpg
    mycone.jpg
    40.4 KB · Views: 453
Last edited:
Question: A clock's minute hand has length 4 and its hour hand has length 3. What is the distance between the tips at the moment when it is increasing most rapidly?(Putnam Exam Question) Answer: Making assumption that both the hands moves at constant angular velocities, the answer is ## \sqrt{7} .## But don't you think this assumption is somewhat doubtful and wrong?

Similar threads

  • · Replies 3 ·
Replies
3
Views
2K
  • · Replies 4 ·
Replies
4
Views
2K
  • · Replies 2 ·
Replies
2
Views
2K
  • · Replies 7 ·
Replies
7
Views
2K
Replies
2
Views
3K
  • · Replies 4 ·
Replies
4
Views
2K
Replies
5
Views
2K
  • · Replies 9 ·
Replies
9
Views
2K
  • · Replies 1 ·
Replies
1
Views
2K
  • · Replies 1 ·
Replies
1
Views
3K